A university should not be entitled to patent the inventions of its faculty members. Universities, as guarantors of i...

Boram on January 11, 2019

Answer choice (d)

Could you please explain why answer coice (d) is the correct answer? Thank you!

Reply
Create a free account to read and take part in forum discussions.

Already have an account? log in

Ravi on January 11, 2019

@Boram,

Happy to help.

The stimulus concludes that a university should not be entitled to
patent the inventions of its faculty members. The support for this
conclusion is that patents equate to promoting a motive to suppress
information/ideas, and this is incompatible with a university's
obligation to promote the free flow of ideas.

We're asked to identify an assumption the argument makes. This is a
strengthen with a necessary premise question. We're looking for an
answer choice that, if false, would make the argument fall apart. In
other words, we're looking for an assumption that must be made by the
argument in order for it to stand a chance.

The stimulus states that a university "has a motive to suppress
information about a potentially valuable discovery until the patent
for it has been secured. Clearly, suppressing information concerning
such discoveries is incompatible with the university's obligation to
promote the free flow of ideas."

There's a big gap there. Just because a university has a motive to do
something doesn't mean that it will act on it. The author is assuming
that they do act on this motive because that's the only way his
argument can make any sense.

Not all strengthen with a necessary premise questions are like this,
where we can see the necessary assumption being made before even
getting to the answers. So, in looking at the answers, we'll negate
each to see if it wrecks our argument.

Answer A is incorrect because its negation does not wreck the
argument. If universities are not the only institutions that have an
obligation to guarantee intellectual freedom, the argument is not
destroyed. This is not a necessary premise.

Answer B is incorrect because if less than half of inventions by
university faculty members would be profitable if patented, the
argument still stands and is not necessarily affected.

Answer C is incorrect because if the publication of reports on
research is not the only practical way to disseminate information
concerning new discoveries, the argument can still stand and is not
wrecked.

Answer D is correct because it states the assumption we identified in
our analysis as being one that is made in the argument. D states that
universities have a motive to suppress information concerning
discoveries by their faculty members will occasionally act on that
motive. D's negation, universities that have a motive to suppress
information concerning discoveries by their faculty members will never
act on that motive, would make the argument fall apart.
Remember the gap we saw in the argument between having a motive and
acting on it? D picks up on this gap; it's true that the argument
assumes that the universities will at least sometimes act on their
motive.

Answer E is incorrect because the argument is concerned with whether
or not universities should be entitled to patent inventions, not
whether or not a faculty member would patent his or her invention.
This answer choice is not assumed by the argument and is incorrect.

Does this make sense? Let us know if you have any additional questions!